$\begingroup$
$\endgroup$
is the scheduling strategy round robin preemptive?
Lets say the quantum is 3 and a process needs 10 to finish but after the first three there is no new process so it continues to run, at t=5 a second process appears, will the first process be stopped since it already used more than 3?
Or does the first process get a new quantum of 3 at t=3 when it is immediately requeued as it is the only one.